Club CIE

Bonjour à ceux et celles que le calcul exact d’intégrales les passionne. Si vous voulez participer au club, donnez svp une réponse complète et non pas des indications comme le font toujours FDP , YvesM et gilles benson
Pour le partage, vous pouvez proposer ici même des intégrales à calculer mais après que la dernière soit résolue complètement et rédigé par un bénévole qui propose une solution. Si l'idée est mauvaise on oublie tout.

Pour commencer je propose une des classique, peut être à chercher une méthode moins connue
1-$ \int_0^{\pi} \frac{x\sin x}{1-\cos x} dx$
Le 😄 Farceur


«13

Réponses

  • \begin{align}J&=\int_0^{\frac{\pi}{2}} \frac{x\sin x}{1-\cos x}dx+\underbrace{\int_{\frac{\pi}{2}}^\pi \frac{x\sin x}{1-\cos x}dx}_{y=\pi-x}\\
    &=\underbrace{\pi \int_0^{\frac{\pi}{2}} \frac{\sin y}{1+\cos y}dy}_{x=\cos y}+\underbrace{\int_0^{\frac{\pi}{2}}\frac{2x}{\tan x}dx}_{\text{IPP}}\\
    &=\pi \int_0^1 \frac{1}{1+x}dx+\Big[2x\log(\sin x)\Big]_0^{\frac{\pi}{2}}-2\underbrace{\int_0^{\frac{\pi}{2}} \log(\sin x)dx}_{=\frac{1}{2}\pi\ln 2}\\
    &=\boxed{2\pi\ln 2}
    \end{align}
  • Très efficace comme méthode et très rapide. Une autre suggestion?
    Le 😄 Farceur


  • 2) $\displaystyle \int_0^1 \dfrac{\ln\left(\frac{1}{2}x+\sqrt{1+\frac{1}{4}x^2}\right)}{x}dx$
  • j'ai une idée d'intégrale à calculer pour la troisième intégrale
    Je suis donc je pense 
  • Pour l'intégrale numéro 1 on peut faire un peu plus simple:

    \begin{align}J&=\int_0^{\pi} \frac{x\sin x}{1-\cos x} dx\\
    &\overset{\text{IPP}}=2\left[x\ln\left(\sin\left(\frac{x}{2}\right)\right)\right]_0^{\pi}-2\int_0^{\pi} \ln\left(\sin\left(\frac{x}{2}\right)\right)\\
    &\overset{y=\frac{x}{2}}=-4\int_0^{\frac{\pi}{2}} \ln(\sin y)dy\\
    &=\boxed{2\pi\ln 2}
    \end{align}

    NB:
    On peut appliquer le "protocole" suivant face à une intégrale de la forme $\int_a^b xf(x)dx$.B-)-
    1) on regarde si le changement de variable $y=a+b-x$ donne quelque chose d'intéressant.
    (dans notre cas d'espèce, pas vraiment)
    2) Si le point 1) n'a rien donné de probant essayer une intégration par parties pour faire disparaître le $x$.
    (Ce qui fonctionne dans notre cas d'espèce)
  • tu es vraiment trop fort en maths Fdp
    Je suis donc je pense 
  • Quentino37: Crois-tu que j'ai fait à la main un calcul de primitive? (j'aurais pu cela dit, pas très compliqué dans le cas d'espèce mais il faut ensuite dégainer une identité trigonométrique).
    Je me doutais seulement, au vu du résultat, qu'il devait exister une primitive simple. J'ai demandé à Wolfy de faire le calcul pour moi.
    Dans ce type de calcul il y a des recettes générales qui peuvent marcher. Tu commences par les appliquer en espérant que cela va permettre de finir le calcul. Et si cela ne fonctionne pas, il faut être plus imaginatif. B-)-

    PS:
    Mes outils sont:
    Pour le calcul de primitives:
    Maxima, Wolfy

    GP PARI: valeurs approchées et, parfois, on peut deviner la valeur exacte d'une intégrale par la fonction lindep
    Cette dernière fonction permet de savoir si des réels sont dépendants entre eux c'est à dire si $\alpha_1,\alpha_2,..,\alpha_n$ sont des réels, la fonction lindep renvoie des coefficients entiers $\lambda_1,....,\lambda_n$ tels que $\lambda_1.\alpha_1+...+\lambda_n.\alpha_n$ est proche de $0$.
    Dans la pratique les nombres $\alpha_1,\alpha_2,..,\alpha_n$ sont des nombres décimaux, et si les valeurs $\lambda_i$ ne sont pas trop grandes alors on a probablement trouvé une dépendance entre ces nombres réels.
  • Pour la 1) on peut aussi faire compliqué voir les deux liens
    Pour la 2) voila où j'en suis
    $\displaystyle \int_0^1 \dfrac{\ln\left(\frac{1}{2}x+\sqrt{1+\frac{1}{4}x^2}\right)}{x}dx=\displaystyle \int_0^{\frac 12} \dfrac{\ln\left(x+\sqrt{1+x^2}\right)}{x}dx $ et par ipp cela donne
    $=\big[\log x\log(x+\sqrt{1+x^2})\big]_0^{\frac 12}-\int_{0}^{{\frac 12}}\dfrac{\log x}{\sqrt {1+x^2}}dx$

    Je suis en face d'une méchante intégrale $$\int_{0}^{{\frac 12}}\dfrac{\log x}{\sqrt {1+x^2}}dx$$
    Le 😄 Farceur


  • Gebrane:

    Le calcul de l'intégrale 1) qui est suggéré dans le problème que tu as mis en pièce jointe ne suppose pas la connaissance de la valeur de l'intégrale $\displaystyle \int_0^{\frac{\pi}{2}}\ln(\sin x)dx$ comme je l'ai supposée.

    Pour ton calcul de l'intégrale 2, tu ne vois vraiment pas comment continuer? (je ne sais pas si cela va marcher mais cela vaut le coup d'essayer). B-)-
  • peut être en posant $y=i\sin(x)$
    Je suis donc je pense 
  • Un changement de variable avec un facteur qui est un nombre complexe?
    Si on fait ce que tu dis quelles seront les nouvelles bornes d'intégration? :-D
  • Quentino37: jamais entendu parler des fonctions hyperboliques?
  • FDP, je réfléchie ! mais wolphi me givre http://www.wolframalpha.com/input/?i=Integrate[Log[x]/Sqrt[1+x^2],{x}]

    Il y a des Li_2

    Quentino37 ici dans ce fil, on ne donne pas des indications: on donne une solution complété ou ce qu'on a réussi à faire
    Le 😄 Farceur


  • à oui! merci Fdp! j'avais oublié les fonctions hyperboliques!
    En posant $x=\sinh(y)$ du coup!

    PS: j'ai écrit le message en même temps que le dernier, j'ai lu le message après
    Je suis donc je pense 
  • On essaie de respecter le protocole de Gebrane.
    Dès qu'on a une solution (qu'on a travaillée suffisamment) on l'écrit ou autrement on s'abstient de poster. Mais j'imagine qu'on est autorisé à poster un commentaire qui fait suite à une solution. dans le même message. B-)-
  • Bonjour,

    Pour la 1), $\displaystyle x \leadsto y$ avec $\displaystyle x=2 y$, puis simplification : $\displaystyle \sin(2y ) =2 \sin y \cos y$ et $\displaystyle 1-\cos (2y) =2 \sin^2(y)$, puis intégration par parties de $\displaystyle \int_0^{\pi/2} y {\sin y \over \cos y} dy$ pour aboutir au résultat : $\displaystyle 2 \pi \ln 2.$

    Pour la 2), intégration par parties avec terme intégré nul puisque $\displaystyle {x \ln x \over 2} \to 0, (x \to 0)$, puis $\displaystyle x \leadsto u$ avec $\displaystyle x=2u$ pour obtenir $\displaystyle -\int_0^{1/2} {\ln (2 u) \over \sqrt{1+u^2}} du$ et après séparation en deux intégrales par $\displaystyle \ln(2u) = \ln 2 + \ln u$, $\displaystyle u \leadsto v$ avec $\displaystyle u = \sinh v$ pour aboutir à $\displaystyle -\ln 2\, \mathrm{argsinh}(1/2) + {\pi^2 \over 20} + 2 \ln 2 (\mathrm{argtanh}(2-\sqrt{5}) + \ldots)$ je vous laisse trouver le terme manquant.

    Si vous trouvez $\displaystyle \ln \phi$, je suis impressionné.

    Ne reste qu'à simplifier car $\displaystyle \mathrm{argsinh}(1/2) =\ln \phi$ et $\displaystyle \mathrm{argtanh}(2-\sqrt{5})={1 \over 2} \ln {3-\sqrt{5} \over \sqrt{5}-1}.$

    Et, à la fin, on élimine tout car $\ln 1 = 0$ et on trouve le résultat : $\displaystyle{ \pi^2 \over 20}.$
  • Oui, ces règles ont pour but de ne pas encombrer le fil, on écrit que le nécessaire utile.
    Des commentaires utiles oui, nos essaies oui , une solution complète oui .

    Je ne suis pas autoritaire (ici le président honoraire du club est FDP ) , Les règles sont faites pour être enfreintes :-D
    Le 😄 Farceur


  • Voilà pour la troisième intégrale que je propose.

    Calculez en fonction de $\lambda$ (qui est strictement positif) et de $n$ : \[\int_{\lambda}^{\lambda+1}D_ {n}(x)dx,
    \] avec $D_{n}(x)$ une fonction telle que :
    1) $D_{n}(x)$ est strictement croissante,
    2) $D_{n}(1)=0$,
    3) $D_{n}(x+1)=D_{n}(x)+\frac{1}{x^{n}}.$
    $D_{n}(x)$ est définie pour tout $n$ strictement supérieur à 1 et $x$ strictement positif.
    ($n$ ; $x$ ; et $\lambda$ sont des réels)

    J’espère ne pas m'être trompé dans l'énoncé ! (ce qui est fort possible).
    Je suis donc je pense 
  • Quentino37:

    A priori je ne vois rien qui permette de s'assurer que l'intégrale considérée a un sens.
  • elle se calcule, enfin je crois(j'ai relu 3 fois mes calculs)
    Je suis donc je pense 
  • Tu n'oublies pas une condition de continuité sur les fonctions $D_n$?
  • il me semble que ça vient automatiquement car la fonction est strictement croissante... et combiné au reste de sa définition... :)
    Je suis donc je pense 
  • Sur des fonctions qui ne sont pas suffisamment régulières (dérivables, continues) je ne peux/sais pas garantir que les méthodes de transformation d'une intégrales sont valides.
  • la fonction est assez régulière: dérivable, continue,... :)
    (enfin, si je me suis pas planté encore une fois)
    Je suis donc je pense 
  • Un intéressant document sur l'intégrabilité d'une intégrale au sens de Riemann.
    Dans ce document on trouve le théorème suivant (théorème 4.5)
    Toute fonction monotone et bornée sur un intervalle fermé borné est intégrable au sens de Riemann.

    NB: dans notre cas d'espèce, du fait de la propriété 3) la fonction considérée est bornée.
  • Heu... non pas vraiment... La fonction n'est pas borné...
    Pour tout M, il existe un $\epsilon$ tel que $D_{n}(\epsilon)<M$
    En revanche, la fonction est bornée sur tout les intervalles $]\lambda, \lambda+1[$ avec $\lambda>0$
    Ou plus précisément sur pour tout $\epsilon>0$ alors la fonction est borné sur $[\epsilon, \infty]$
    Je suis donc je pense 
  • Quentino37: je voulais dire bornée sur l'intervalle $[\lambda,\lambda+1]$

    D'après la propriété 3) on a pour tout $n$ entier naturel non nul, $\displaystyle D_n(\lambda+1)=D_n(\lambda)+\frac{1}{\lambda^n}$

    Ce qui fait que puisque les fonctions $D_n$ sont croissantes on a pour tout $\displaystyle x\in [\lambda,\lambda+1], D_n(\lambda) \leq D_n(x)\leq D_n(\lambda)+\frac{1}{\lambda^n}$
  • Par ailleurs, on montre par récurrence que pour tout $n$ entier naturel et tout $m$ entier naturel strictement plus grand que $1$ que:
    $\displaystyle D_n(m)=\sum_{k=1}^{m-1} \dfrac{1}{k^n}$.

    Démonstration:
    1)$\displaystyle D_n(2)=D_n(1)+\dfrac{1}{1^n}=1$ Or, $\displaystyle \sum_{k=1}^{2-1} \dfrac{1}{k^n}=1$
    2) $\displaystyle D_n(m+1)=D_n(m)+\frac{1}{m^n}=\sum_{k=1}^{m-1} \dfrac{1}{k^n}+\frac{1}{m^n}=\sum_{k=1}^{m} \dfrac{1}{k^n}$.
    On dirait bien que $\displaystyle \lim_{m\rightarrow \infty}D_n(m)=\zeta(n)$

    Or, $\displaystyle \lim_{n\rightarrow \infty}\zeta(n)=1$.
  • Bonjour

    dans l'esprit que souhaite notre ami gebrane je propose 3 intégrales numériques convergentes
    dont le résultat s'exprime avec des constantes classiques :

    $\displaystyle A = \int_0^1\frac{1-e^{-x}}{x}dx$ 3 constantes sont nécessaires ,
    j'ai une solution qui passe par le développement en série de la fonction intégrande

    $\displaystyle B = \int_0^{\frac{\pi}{2}}\sqrt{\tan x}\,dx$ 1 constante suffira ;
    j'ai une solution qui passe par les applications intégrales des séries de Riemann

    $\displaystyle C= \int_0^{+\infty}\frac{\ln x}{\cosh^2x}dx$ (avec chx le cosinus hyperbolique de x) 2 constantes sont nécessaires,
    ma solution passe par la fonctions eulériennes Béta et Gamma

    il est probable qu'il existe d'autres méthodes pour trouver ces résultats d'intégrales numériques.
    Cordialement
  • \begin{align}B&=\int_0^{\frac{\pi}{2}}\sqrt{\tan x}dx\\
    &\overset{y=\sqrt{\tan x}}=\int_0^\infty \frac{2y^2}{1+y^4}dy\\
    &\overset{z=\frac{1}{y}}=\int_0^\infty \frac{2}{1+y^4}dz\\
    2B&=2\int_0^\infty \frac{1+y^2}{1+y^4}dy\\
    B&=\int_0^\infty \frac{1+\frac{1}{y^2}}{\left(y-\frac{1}{y}\right)^2+2}dy\\
    &\overset{z=y-\frac{1}{y}}=\int_{-\infty}^\infty \frac{1}{z^2+2}dz\\
    &=\left[\frac{1}{\sqrt{2}}\arctan\left(\frac{x}{\sqrt{2}}\right)\right]_{-\infty}^\infty\\
    &=\boxed{\dfrac{\pi}{\sqrt{2}}}
    \end{align}
  • FDP, je reviens à la 2) avec cette méthode, crois-tu que ça mène au résultat rapidement ?

    edit fautes sur les bornes signalé par FDP, à remplacer 0 par 1 et $1+\sqrt 2$ par $1/2+\sqrt{1+1/4}$
    Le 😄 Farceur


  • bonjour FdP

    ton résultat de l'intégrale B est le bon, bien-sûr

    le résultat est le même avec l'intégrale $\int_0^{\frac{\pi}{2}}\frac{dx}{tan(x)}$

    et d'une façon générale avec - 1 < x < 1 l'intégrale
    $$\int_0^{\frac{\pi}{2}}tan^x(t)dt = \frac{\pi}{2cos\frac{\pi}{2}x}$$ fonction paire en x

    je signale aussi que la primitive de $\sqrt{tan(t)}$ avec $0 < x < \frac{\pi}{2}$

    $\int_0^x\sqrt{tan(t)}dt$ s'exprime en fonction de tan(x)

    cordialement
  • Fdp: Tu as raison, quand m tend vers l'infini, $D_{n}(m)$ tend vers $\zeta(n)$
    Je suis donc je pense 
  • Gebrane:

    Il y a des erreurs dans ton calcul (les bornes d'intégration après changement de variable)
  • Bonjour

    Calculer $$\int_{0}^{+oo}\frac{\arctan(x)}{x^5+1} dx$$
  • pour le symbole $\infty$ c'est \infty en latex (tu)
    Je suis donc je pense 
  • Oui FDP, j’étais distrait pour les bornes ( faute d'une relecture)
    Le 😄 Farceur


  • Bonjour, je reviens sur $I = \displaystyle \int_0^1 \dfrac{\ln\left(x/2 + \sqrt{1 + x^2/4} \right) }{x} \mathrm dx$ :
    1) on connaît $\sinh^{-1}(x/a) = \ln\left(x/a + \sqrt{1 + x^2/a^2} \right)$ et sa dérivée: $ \dfrac{1}{a\sqrt{1+x^2/a^2}}$.
    On voit que $\sinh^{-1}(1/2) = \ln\left(1/2 + \sqrt{1 + 1/4} \right) = \ln \phi$.
    2) on connaît la fonction dilogarithme: $ Li_2(x) = \displaystyle \sum_{n\geq 1} \dfrac{x^n}{n^2}$ qui vérifie:
    * $Li_2(1) = \dfrac{\pi^2}{6}$.
    * $ Li_2(1/\phi^2) = \dfrac{\pi^2}{15} - \frac{1}{4}\left(\ln(1/\phi^2)\right)^2 = \dfrac{\pi^2}{15} - \left(\ln(\phi^2)\right)^2$.
    On rappelle que $\phi = \dfrac{1+\sqrt{5}}{2}$ et $1/\phi^2 = \dfrac{3-\sqrt{5}}{2}$.
    On voit que $\sinh^{-1}(1/2) = \ln\left(1/2 + \sqrt{1 + 1/4} \right) = \ln \phi$

    3) Dans cette intégrale, on effectue une intégration par parties en posant $U= \ln\left(x/2 + \sqrt{1 + x^2/4} \right)$ et $V' = 1/x$, soit: $U'= \dfrac{1}{2\sqrt{1+x^2/4}}$ et $V = \ln x$, soit: $\int UV' = UV - \int U'V$, ce qui donne avec les bornes :
    $$
    I = \left [ \ln x \ln\left(x/2 + \sqrt{1 + x^2/4} \right) \right]_0^1 - \displaystyle \int_0^1 \dfrac{\ln x}{2\sqrt{1+x^2/4}} \mathrm dx.

    $$ Le crochet disparaît car $ \ln x \ln\left(x/2 + \sqrt{1 + x^2/4} \right) = O(x\ln x)$ en zéro.
    Donc, $I = - \displaystyle \int_0^1 \dfrac{\ln x}{2\sqrt{1+x^2/4}} \mathrm dx$ et on effectue le changement de variable $x =2u$, les nouvelles bornes étant $0$ et $1/2$ avec $ dx = 2du$, soit:
    $$I = - \displaystyle \int_0^{1/2} \dfrac{\ln 2 + \ln u}{\sqrt{1+u^2}} \mathrm du.

    $$ On voit : $ \displaystyle \int_0^{1/2} \dfrac{\ln 2 }{\sqrt{1+u^2}} \mathrm du = \ln 2\ln \phi$. On évalue $J = - \displaystyle \int_0^{1/2} \dfrac{ \ln u}{\sqrt{1+u^2}} \mathrm du$ à l'aide du changement de variable $ \sinh u = v$, soit $ \mathrm dv = \dfrac{ 1}{\sqrt{1+u^2}} \mathrm du$, les nouvelles bornes étant $0$ et $\ln \phi$ :
    $$
    J = - \displaystyle \int_0^{\ln \phi} \ln \left( \dfrac{e^v - e^{-v}}{2} \right) \mathrm dv.

    $$ On écrit : $ \ln \left( \dfrac{e^v - e^{-v}}{2} \right) = - \ln 2 + \ln \left( e^v(1 - e^{-2v}) \right) = -\ln 2 + v + \ln \left( 1 - e^{-2v} \right) $.
    On intègre :
    $$
    J = \ln 2 \ln \phi - \frac{1}{2} (\ln \phi)^2 - \displaystyle \int_0^{\ln \phi} \ln \left( 1 - e^{-2v} \right) \mathrm dv .

    $$ 4) on évalue l'intégrale restante en utilisant le développement $ -\ln (1-x) = \displaystyle \sum_{n\geq1} \dfrac{x^n}{n}$ si $|x| < 1$ et on note que $0 < \ln \phi < 1 $ : l'interversion de sommation se justifie car tout est positif ou bien en utilisant le reste du développement en série utilisé qui s'obtient en intégrant $ 1/(1-x) = \sum_0^N x^n + x^{N+1}/(1-x)$
    et là un miracle arrive...
    $$
    - \ln \left( 1 - e^{-2v} \right) \; = \; \displaystyle \sum_{n\geq 1} \frac{1}{n} e^{-2nv}.

    $$ Soit
    $$ \int_0^{\ln \phi} - \ln \left( 1 - e^{-2v} \right) \mathrm dv \; = \; \sum_{n\geq 1} \int_0^{\ln \phi} \frac{1}{n} e^{-2nv} \mathrm dv
    $$ avec
    $$ \int_0^{\ln \phi} \frac{1}{n} e^{-2nv} \mathrm dv \; = \; \left [ \frac{1}{2n^2} \left( e^{-2n v}\right) \right ]_0^{\ln \phi} \; = \; \frac{1}{2n^2} \left( 1 - e^{-2n \ln \phi }\right) .

    $$ Comme $e^{-2n \ln \phi } = (1/ \phi ^2 )^n$, on arrive à $J = \ln 2 \ln \phi - \frac{1}{2} (\ln \phi)^2 + \frac{1}{2}Li_(1) - \frac{1}{2} Li_2 \left(1/ \phi ^2 \right) $.
    C'est maintenant fini puisque $ \frac{1}{2}Li_(1) = \pi^2 /12$ et $\frac{1}{2} Li_2 \left(1/ \phi ^2 \right) = \pi ^2/ 30 - \frac{1}{2} \left(\ln(\phi^2)\right)^2$.
    En revenant à $I$, on obtient $ I = \pi ^2 /20$.

    Référence: Dilogarithms and associated functions de L. Lewin chez Macdonald (1958).
    A demon  wind propelled me east of the sun
  • Bonsoir, disons que j'ai commencé à trouver ce problème intéressant quand j'ai vu apparaître $\ln \phi$ et qu'il avait été fait mention du dilogarithme qui est une de mes marottes.:-)
    A demon  wind propelled me east of the sun
  • Merci gilles benson pour ta rédaction détaillée, c'est très formatrice
    Le 😄 Farceur


  • Ben, j'avais cru comprendre que j'avais été un peu lapidaire...

    Question : sachant que $Li_2(z) = \displaystyle -\int_0^z \dfrac{\ln(1-x)}{x} \mathrm dx$, prouver que si $n \in \mathbb{N}^*$ et $\zeta = \exp \big(\frac{2\pi i}{n} \big)$ :
    $$
    \frac{1}{n} Li_2(z^n ) \; = \; \displaystyle \sum_{k=0}^{n-1} Li_2(\zeta ^k z).$$
    A demon  wind propelled me east of the sun
  • Il vaut peut-être mieux ici prendre cette définition $\displaystyle \text{Li}_2(a)=-a\int_0^1 \dfrac{\ln x}{1-ax}dx$ avec $a$ complexe non nul tel que $\dfrac{1}{a}$ ne soit pas dans l'intervalle $[0,1]$, ou $a=1$, ou $a=0$.
  • Une recente apparition du dilogarithme dans arXiv 2104-05510.
  • @P peux-tu donner le titre et le(s) auteurs de l’article sur le dilogarithm car quand je clique sur le lien t’as mis ça ne marche pas.Merci

    @P ça marche maintenant merci

    Il s’agit de l’article Duality of exponential families: the real case, from large deviations to dilogarithm par Gérard Letac
  • http://arxiv.org/abs/2104.05510




    Ils sont chatouilleux, j'avais mis un tiret au lieu d'un point....
  • Evaluation de $\displaystyle \text{Li}_2\left(\frac{1}{\varphi}\right)$ avec $\displaystyle \varphi=\frac{1+\sqrt{5}}{2}$
    \begin{align}J&=\int_0^{\frac{1}{\varphi}}\frac{\ln x}{1-x}dx\\
    &\overset{u=\frac{1}{1+u}}=\int_{\frac{1}{\varphi}}^1 \frac{\ln\left(\frac{1-u}{u}\right)}{u(2u-1)}du\\
    &=-\int_{\frac{1}{\varphi}}^1 \frac{\ln\left(\frac{1-u}{u}\right)}{u}du-2\int_{\frac{1}{\varphi}}^1 \frac{\ln\left(\frac{1-u}{u}\right)}{1-2u}du\\
    &=-\underbrace{\int_{\frac{1}{\varphi}}^1 \frac{\ln\left(1-u\right)}{u}du}_{\text{IPP}}-\frac{1}{2}\ln^2\varphi-2\underbrace{\int_{\frac{1}{\varphi}}^1 \frac{\ln\left(\frac{1-u}{u}\right)}{1-2u}du}_{z=\frac{1-u}{u}}\\
    &=\left(2\ln^2\varphi+\frac{1}{6}\pi^2+\int_0^{\frac{1}{\varphi}}\frac{\ln u}{1-u}du\right)-\frac{1}{2}\ln^2\varphi+\left(\int_0^{\frac{1}{\varphi}}\frac{\ln z}{1-z}dz+\int_0^{\frac{1}{\varphi}}\frac{\ln z}{1+z}dz\right)\\
    &=\frac{3}{2}\ln^2\varphi+\frac{1}{6}\pi^2+2\int_0^{\frac{1}{\varphi}}\frac{\ln z}{1-z}dz+\int_0^{\frac{1}{\varphi}}\frac{\ln z}{1+z}dz
    \end{align}
    Donc:
    \begin{align}J&=-\frac{3}{2}\ln^2\varphi-\frac{1}{6}\pi^2-\int_0^{\frac{1}{\varphi}}\frac{\ln z}{1+z}dz\\
    \int_0^{\frac{1}{\varphi}}\frac{\ln z}{1+z}dz&=J-\int_0^{\frac{1}{\varphi}}\frac{2z\ln z}{1-z^2}dz\\
    &\overset{u=1-z^2}=J-\frac{1}{2}\int_{\frac{1}{\varphi}}^1 \frac{\ln(1-u)}{u}du\\
    &\overset{\text{IPP}}=J+\ln^2 \varphi-\frac{1}{2}\int_{\frac{1}{\varphi}}^1\frac{\ln u}{1-u}du\\
    &=J+\ln^2 \varphi+\frac{1}{12}\pi^2+\frac{1}{2}J\\
    &=\frac{3}{2}J+\ln^2 \varphi+\frac{1}{12}\pi^2
    \end{align}
    Donc \begin{align}\boxed{J=-\ln^2\varphi-\dfrac{\pi^2}{10}}\end{align}
    Par ailleurs,
    \begin{align}\text{Li}_2\left(\frac{1}{\varphi}\right)&=-\frac{1}{\varphi}\int_0^1 \frac{\ln x}{1-\frac{1}{\varphi}x}dx\\
    &\overset{u=\frac{x}{\varphi}}=-\int_0^{\frac{1}{\varphi}}\frac{\ln\left(\varphi u\right)}{1-u}du\\
    &=-J-\ln\varphi\int_0^{\frac{1}{\varphi}}\frac{1}{1-u}du\\
    &=-J-2\ln^2\varphi
    \end{align}
    Donc: \begin{align}\boxed{\text{Li}_2\left(\dfrac{1}{\varphi}\right)=\dfrac{1}{10}\pi^2-\ln^2\varphi}\end{align}
    NB:
    \begin{align}\dfrac{1}{1+\dfrac{1}{\varphi}}&=\dfrac{1}{\varphi}\\
    \dfrac{1-\dfrac{1}{\varphi}}{\dfrac{1}{\varphi}}&=\dfrac{1}{\varphi}\\
    1-\dfrac{1}{\varphi^2}&=\dfrac{1}{\varphi}
    \end{align}
  • Bon, moi je triche car je dispose de documents supérieurs.

    1) on prouve que: $\frac{1}{2} Li_2(x^2) = Li_2(x) + Li_2(-x)$ (voir la question posée plus haut).
    2) on prouve que: $Li_2(x) + Li_2 \Big(\dfrac{-x}{1-x} \Big) = -\frac{1}{2} \ln ^2 (1-x) $ (dériver).
    3) on prouve: $ Li_2(x) + Li_2(1-x) = \frac{\pi ^2}{6} - \ln (x) \ln (1-x) $.
    On additionne 1) et 2):
    $$
    Li_2 \Big(\dfrac{-x}{1-x} \Big) +\frac{1}{2} Li_2(x^2) - Li_2(-x)= -\frac{1}{2} \ln ^2 (1-x) .

    $$ On prend dans cette égalité $x = -1/\phi$ solution de $x^2 -x-1=0$, ce qui donne puisque $1 +\phi = \phi ^2$ :
    $$
    \frac{3}{2} Li_2 \Big(\dfrac{1}{\phi ^2} \Big) -Li_2 \Big(\dfrac{1}{\phi } \Big) = -\frac{1}{2} \ln ^2 (\phi)


    $$ Dans la relation 3), on prend $x = \frac{1}{\phi ^2}$ :
    $$
    Li_2\Big(\frac{1}{\phi ^2}\Big) + Li_2\Big(\frac{1}{\phi} \Big) = \frac{\pi ^2}{6} - \ln \Big(\frac{1}{\phi ^2}\Big) \ln \Big(\frac{1}{\phi }\Big).

    $$ On obtient un système de deux équations que l'on résout: $Li_2 \Big(\dfrac{1}{\phi } \Big) = \frac{\pi ^2}{10} - \ln ^2 (\phi)$.
    A demon  wind propelled me east of the sun
  • Merci FDP et g.b
    Une question délicieuse B-)-

    7) Montrer que $$ \int_0^1 \int_0^1 \frac {1}{\phi - xy}\, dx\, dy=\text{Li}_2\left(\frac {1}{\phi}\right)$$
    Le 😄 Farceur


  • vous n'avez toujours pas résolu l'intégrale que j'avais proposé! ;) (tu)
    Je suis donc je pense 
Connectez-vous ou Inscrivez-vous pour répondre.